Limit of weak equivalences in a Bousfield localization










5














Let $M$ be a model category and $C$ a class of maps in it, and assume the left Bousfield localization $L_CM$ exists. Suppose we are given sequences of maps $(p_n+1: X_n+1to X_n), (q_n+1: Y_n+1to Y_n), (f_n: X_nto Y_n), n=0, 1,ldots$ with $q_n+1f_n+1=f_np_n+1$, so we get a ladder of commutative squares. If each $p_n$ is a fibration of fibrants in $M$, each $q_n$ is a fibration of fibrants in $L_CM$, and each $f_n$ is a weak equivalence in $L_CM$, can we conclude that the limit map $lim f_n$ is also a weak equivalence in $L_CM$?



For the notion of left Bousfield localization, see Hirschhorn, Model categories and their localizations, chapter 3, 4. See Proposition 15.10.12 in that book for a similar result, my question is by weakening the assumption as well as the conclusion. You may add suitable and reasonable conditions—like simplicial, properness, cofibrantly generated, etc.—if needed.










share|cite|improve this question




























    5














    Let $M$ be a model category and $C$ a class of maps in it, and assume the left Bousfield localization $L_CM$ exists. Suppose we are given sequences of maps $(p_n+1: X_n+1to X_n), (q_n+1: Y_n+1to Y_n), (f_n: X_nto Y_n), n=0, 1,ldots$ with $q_n+1f_n+1=f_np_n+1$, so we get a ladder of commutative squares. If each $p_n$ is a fibration of fibrants in $M$, each $q_n$ is a fibration of fibrants in $L_CM$, and each $f_n$ is a weak equivalence in $L_CM$, can we conclude that the limit map $lim f_n$ is also a weak equivalence in $L_CM$?



    For the notion of left Bousfield localization, see Hirschhorn, Model categories and their localizations, chapter 3, 4. See Proposition 15.10.12 in that book for a similar result, my question is by weakening the assumption as well as the conclusion. You may add suitable and reasonable conditions—like simplicial, properness, cofibrantly generated, etc.—if needed.










    share|cite|improve this question


























      5












      5








      5


      1





      Let $M$ be a model category and $C$ a class of maps in it, and assume the left Bousfield localization $L_CM$ exists. Suppose we are given sequences of maps $(p_n+1: X_n+1to X_n), (q_n+1: Y_n+1to Y_n), (f_n: X_nto Y_n), n=0, 1,ldots$ with $q_n+1f_n+1=f_np_n+1$, so we get a ladder of commutative squares. If each $p_n$ is a fibration of fibrants in $M$, each $q_n$ is a fibration of fibrants in $L_CM$, and each $f_n$ is a weak equivalence in $L_CM$, can we conclude that the limit map $lim f_n$ is also a weak equivalence in $L_CM$?



      For the notion of left Bousfield localization, see Hirschhorn, Model categories and their localizations, chapter 3, 4. See Proposition 15.10.12 in that book for a similar result, my question is by weakening the assumption as well as the conclusion. You may add suitable and reasonable conditions—like simplicial, properness, cofibrantly generated, etc.—if needed.










      share|cite|improve this question















      Let $M$ be a model category and $C$ a class of maps in it, and assume the left Bousfield localization $L_CM$ exists. Suppose we are given sequences of maps $(p_n+1: X_n+1to X_n), (q_n+1: Y_n+1to Y_n), (f_n: X_nto Y_n), n=0, 1,ldots$ with $q_n+1f_n+1=f_np_n+1$, so we get a ladder of commutative squares. If each $p_n$ is a fibration of fibrants in $M$, each $q_n$ is a fibration of fibrants in $L_CM$, and each $f_n$ is a weak equivalence in $L_CM$, can we conclude that the limit map $lim f_n$ is also a weak equivalence in $L_CM$?



      For the notion of left Bousfield localization, see Hirschhorn, Model categories and their localizations, chapter 3, 4. See Proposition 15.10.12 in that book for a similar result, my question is by weakening the assumption as well as the conclusion. You may add suitable and reasonable conditions—like simplicial, properness, cofibrantly generated, etc.—if needed.







      homotopy-theory model-categories bousfield-localization






      share|cite|improve this question















      share|cite|improve this question













      share|cite|improve this question




      share|cite|improve this question








      edited yesterday

























      asked yesterday









      Lao-tzu

      408312




      408312




















          2 Answers
          2






          active

          oldest

          votes


















          3














          No. For a counterexample to your claim, consider the model category M
          of simplicial presheaves on a small site S equipped with the projective
          model structure.
          Its fibrant objects are presheaves of Kan complexes.
          If C is the set of Čech covers of S, then L_C(M) is the local projective
          model structure on simplicial presheaves.
          Its fibrant objects are presheaves of Kan complexes that satisfy homotopy descent.
          A weak equivalence from a fibrant object in M to a fibrant object in L_C(M)
          is a homotopy sheafification map.
          Furthermore, the limit of p and q is a homotopy limit in M,
          so lim f_n is a weak equivalence if and only if the homotopy sheafification
          functor preserves homotopy limits of towers.
          This is false for arbitrary sites.






          share|cite|improve this answer
















          • 1




            What's your def of homotopy (co)limits? I know it's discussed in the last two chapters of Hirschhorn's book by formulas. But his homotopy limits has not fully homotopy invariance—the objects involved in the diagrams should be fibrant in the model structure. But someone else will take functorial fibrant replacement before using the formulas, which I know is weakly equivalent to the right derived functor of $lim$ in nice cases (and someone always use this right derived functor as def of homotopy limits). Do you know what is the most "correct" and/or most accepted def?
            – Lao-tzu
            17 hours ago











          • The difficulty of defining homotopy limits and colimits by universal property is one of the main motivations behind $infty$-categories. Without such a definition, all 'definitions' are actually nontrivially equivalent computations.
            – Harry Gindi
            16 hours ago










          • @Harry Gindi Is the notion of homotopy limits in a model category just a special case of limits for $infty$-categories (when taking the $infty$-category associated to the given model category) or there is some relation you can explain?
            – Lao-tzu
            15 hours ago






          • 1




            @DimitriPavlov I found unwinding the functorial case in the relative category model to be highly involved. It is a large portion of the book of Dwyer-Hirschhorn-Kan-Smith. Usually you can punt the question and pop through to the hammock localization and then define it in terms of hom-wise simplicial holims, but in the Quasicategory case, the non-functorial definition amounts to nothing more than a terminal object of a slice over the diagram.
            – Harry Gindi
            13 hours ago






          • 1




            @HarryGindi: Sure, the unwinding can be involved, but notice that your original claim was not about the complexity of computations, but rather about the complexity of definitions, a different notion.
            – Dmitri Pavlov
            13 hours ago


















          7














          In the language of $infty$-categories, which makes it a bit clearer, this is asking for the reflector (left adjoint) of the inclusion of a reflective subcategory to preserve filtered limits. This isn't true for ordinary categories, and there is also no reason to expect it to be true for $infty$-categories.



          Hirschhorn's Proposition 15.10.12 says that the homotopy limit of a tower of fibrations can be computed as the ordinary limit. Your modification asks for this homotopy limit to be preserved by the reflector (localization functor).






          share|cite|improve this answer
















          • 2




            As for a condition for this to hold, it's true if the $f_n$ are $M$-equivalences, but this makes the statement trivial, since it implies that the $X_n$ are $C$-local and forces the $p_n$ to be local fibrations (by the usual results about left Bousfield localization, being $C$-local is invariant under $M$-equivalence and $M$-fibrations between local objects are also local fibrations.)
            – Harry Gindi
            23 hours ago











          Your Answer





          StackExchange.ifUsing("editor", function ()
          return StackExchange.using("mathjaxEditing", function ()
          StackExchange.MarkdownEditor.creationCallbacks.add(function (editor, postfix)
          StackExchange.mathjaxEditing.prepareWmdForMathJax(editor, postfix, [["$", "$"], ["\\(","\\)"]]);
          );
          );
          , "mathjax-editing");

          StackExchange.ready(function()
          var channelOptions =
          tags: "".split(" "),
          id: "504"
          ;
          initTagRenderer("".split(" "), "".split(" "), channelOptions);

          StackExchange.using("externalEditor", function()
          // Have to fire editor after snippets, if snippets enabled
          if (StackExchange.settings.snippets.snippetsEnabled)
          StackExchange.using("snippets", function()
          createEditor();
          );

          else
          createEditor();

          );

          function createEditor()
          StackExchange.prepareEditor(
          heartbeatType: 'answer',
          autoActivateHeartbeat: false,
          convertImagesToLinks: true,
          noModals: true,
          showLowRepImageUploadWarning: true,
          reputationToPostImages: 10,
          bindNavPrevention: true,
          postfix: "",
          imageUploader:
          brandingHtml: "Powered by u003ca class="icon-imgur-white" href="https://imgur.com/"u003eu003c/au003e",
          contentPolicyHtml: "User contributions licensed under u003ca href="https://creativecommons.org/licenses/by-sa/3.0/"u003ecc by-sa 3.0 with attribution requiredu003c/au003e u003ca href="https://stackoverflow.com/legal/content-policy"u003e(content policy)u003c/au003e",
          allowUrls: true
          ,
          noCode: true, onDemand: true,
          discardSelector: ".discard-answer"
          ,immediatelyShowMarkdownHelp:true
          );



          );













          draft saved

          draft discarded


















          StackExchange.ready(
          function ()
          StackExchange.openid.initPostLogin('.new-post-login', 'https%3a%2f%2fmathoverflow.net%2fquestions%2f319485%2flimit-of-weak-equivalences-in-a-bousfield-localization%23new-answer', 'question_page');

          );

          Post as a guest















          Required, but never shown

























          2 Answers
          2






          active

          oldest

          votes








          2 Answers
          2






          active

          oldest

          votes









          active

          oldest

          votes






          active

          oldest

          votes









          3














          No. For a counterexample to your claim, consider the model category M
          of simplicial presheaves on a small site S equipped with the projective
          model structure.
          Its fibrant objects are presheaves of Kan complexes.
          If C is the set of Čech covers of S, then L_C(M) is the local projective
          model structure on simplicial presheaves.
          Its fibrant objects are presheaves of Kan complexes that satisfy homotopy descent.
          A weak equivalence from a fibrant object in M to a fibrant object in L_C(M)
          is a homotopy sheafification map.
          Furthermore, the limit of p and q is a homotopy limit in M,
          so lim f_n is a weak equivalence if and only if the homotopy sheafification
          functor preserves homotopy limits of towers.
          This is false for arbitrary sites.






          share|cite|improve this answer
















          • 1




            What's your def of homotopy (co)limits? I know it's discussed in the last two chapters of Hirschhorn's book by formulas. But his homotopy limits has not fully homotopy invariance—the objects involved in the diagrams should be fibrant in the model structure. But someone else will take functorial fibrant replacement before using the formulas, which I know is weakly equivalent to the right derived functor of $lim$ in nice cases (and someone always use this right derived functor as def of homotopy limits). Do you know what is the most "correct" and/or most accepted def?
            – Lao-tzu
            17 hours ago











          • The difficulty of defining homotopy limits and colimits by universal property is one of the main motivations behind $infty$-categories. Without such a definition, all 'definitions' are actually nontrivially equivalent computations.
            – Harry Gindi
            16 hours ago










          • @Harry Gindi Is the notion of homotopy limits in a model category just a special case of limits for $infty$-categories (when taking the $infty$-category associated to the given model category) or there is some relation you can explain?
            – Lao-tzu
            15 hours ago






          • 1




            @DimitriPavlov I found unwinding the functorial case in the relative category model to be highly involved. It is a large portion of the book of Dwyer-Hirschhorn-Kan-Smith. Usually you can punt the question and pop through to the hammock localization and then define it in terms of hom-wise simplicial holims, but in the Quasicategory case, the non-functorial definition amounts to nothing more than a terminal object of a slice over the diagram.
            – Harry Gindi
            13 hours ago






          • 1




            @HarryGindi: Sure, the unwinding can be involved, but notice that your original claim was not about the complexity of computations, but rather about the complexity of definitions, a different notion.
            – Dmitri Pavlov
            13 hours ago















          3














          No. For a counterexample to your claim, consider the model category M
          of simplicial presheaves on a small site S equipped with the projective
          model structure.
          Its fibrant objects are presheaves of Kan complexes.
          If C is the set of Čech covers of S, then L_C(M) is the local projective
          model structure on simplicial presheaves.
          Its fibrant objects are presheaves of Kan complexes that satisfy homotopy descent.
          A weak equivalence from a fibrant object in M to a fibrant object in L_C(M)
          is a homotopy sheafification map.
          Furthermore, the limit of p and q is a homotopy limit in M,
          so lim f_n is a weak equivalence if and only if the homotopy sheafification
          functor preserves homotopy limits of towers.
          This is false for arbitrary sites.






          share|cite|improve this answer
















          • 1




            What's your def of homotopy (co)limits? I know it's discussed in the last two chapters of Hirschhorn's book by formulas. But his homotopy limits has not fully homotopy invariance—the objects involved in the diagrams should be fibrant in the model structure. But someone else will take functorial fibrant replacement before using the formulas, which I know is weakly equivalent to the right derived functor of $lim$ in nice cases (and someone always use this right derived functor as def of homotopy limits). Do you know what is the most "correct" and/or most accepted def?
            – Lao-tzu
            17 hours ago











          • The difficulty of defining homotopy limits and colimits by universal property is one of the main motivations behind $infty$-categories. Without such a definition, all 'definitions' are actually nontrivially equivalent computations.
            – Harry Gindi
            16 hours ago










          • @Harry Gindi Is the notion of homotopy limits in a model category just a special case of limits for $infty$-categories (when taking the $infty$-category associated to the given model category) or there is some relation you can explain?
            – Lao-tzu
            15 hours ago






          • 1




            @DimitriPavlov I found unwinding the functorial case in the relative category model to be highly involved. It is a large portion of the book of Dwyer-Hirschhorn-Kan-Smith. Usually you can punt the question and pop through to the hammock localization and then define it in terms of hom-wise simplicial holims, but in the Quasicategory case, the non-functorial definition amounts to nothing more than a terminal object of a slice over the diagram.
            – Harry Gindi
            13 hours ago






          • 1




            @HarryGindi: Sure, the unwinding can be involved, but notice that your original claim was not about the complexity of computations, but rather about the complexity of definitions, a different notion.
            – Dmitri Pavlov
            13 hours ago













          3












          3








          3






          No. For a counterexample to your claim, consider the model category M
          of simplicial presheaves on a small site S equipped with the projective
          model structure.
          Its fibrant objects are presheaves of Kan complexes.
          If C is the set of Čech covers of S, then L_C(M) is the local projective
          model structure on simplicial presheaves.
          Its fibrant objects are presheaves of Kan complexes that satisfy homotopy descent.
          A weak equivalence from a fibrant object in M to a fibrant object in L_C(M)
          is a homotopy sheafification map.
          Furthermore, the limit of p and q is a homotopy limit in M,
          so lim f_n is a weak equivalence if and only if the homotopy sheafification
          functor preserves homotopy limits of towers.
          This is false for arbitrary sites.






          share|cite|improve this answer












          No. For a counterexample to your claim, consider the model category M
          of simplicial presheaves on a small site S equipped with the projective
          model structure.
          Its fibrant objects are presheaves of Kan complexes.
          If C is the set of Čech covers of S, then L_C(M) is the local projective
          model structure on simplicial presheaves.
          Its fibrant objects are presheaves of Kan complexes that satisfy homotopy descent.
          A weak equivalence from a fibrant object in M to a fibrant object in L_C(M)
          is a homotopy sheafification map.
          Furthermore, the limit of p and q is a homotopy limit in M,
          so lim f_n is a weak equivalence if and only if the homotopy sheafification
          functor preserves homotopy limits of towers.
          This is false for arbitrary sites.







          share|cite|improve this answer












          share|cite|improve this answer



          share|cite|improve this answer










          answered yesterday









          Dmitri Pavlov

          13k43482




          13k43482







          • 1




            What's your def of homotopy (co)limits? I know it's discussed in the last two chapters of Hirschhorn's book by formulas. But his homotopy limits has not fully homotopy invariance—the objects involved in the diagrams should be fibrant in the model structure. But someone else will take functorial fibrant replacement before using the formulas, which I know is weakly equivalent to the right derived functor of $lim$ in nice cases (and someone always use this right derived functor as def of homotopy limits). Do you know what is the most "correct" and/or most accepted def?
            – Lao-tzu
            17 hours ago











          • The difficulty of defining homotopy limits and colimits by universal property is one of the main motivations behind $infty$-categories. Without such a definition, all 'definitions' are actually nontrivially equivalent computations.
            – Harry Gindi
            16 hours ago










          • @Harry Gindi Is the notion of homotopy limits in a model category just a special case of limits for $infty$-categories (when taking the $infty$-category associated to the given model category) or there is some relation you can explain?
            – Lao-tzu
            15 hours ago






          • 1




            @DimitriPavlov I found unwinding the functorial case in the relative category model to be highly involved. It is a large portion of the book of Dwyer-Hirschhorn-Kan-Smith. Usually you can punt the question and pop through to the hammock localization and then define it in terms of hom-wise simplicial holims, but in the Quasicategory case, the non-functorial definition amounts to nothing more than a terminal object of a slice over the diagram.
            – Harry Gindi
            13 hours ago






          • 1




            @HarryGindi: Sure, the unwinding can be involved, but notice that your original claim was not about the complexity of computations, but rather about the complexity of definitions, a different notion.
            – Dmitri Pavlov
            13 hours ago












          • 1




            What's your def of homotopy (co)limits? I know it's discussed in the last two chapters of Hirschhorn's book by formulas. But his homotopy limits has not fully homotopy invariance—the objects involved in the diagrams should be fibrant in the model structure. But someone else will take functorial fibrant replacement before using the formulas, which I know is weakly equivalent to the right derived functor of $lim$ in nice cases (and someone always use this right derived functor as def of homotopy limits). Do you know what is the most "correct" and/or most accepted def?
            – Lao-tzu
            17 hours ago











          • The difficulty of defining homotopy limits and colimits by universal property is one of the main motivations behind $infty$-categories. Without such a definition, all 'definitions' are actually nontrivially equivalent computations.
            – Harry Gindi
            16 hours ago










          • @Harry Gindi Is the notion of homotopy limits in a model category just a special case of limits for $infty$-categories (when taking the $infty$-category associated to the given model category) or there is some relation you can explain?
            – Lao-tzu
            15 hours ago






          • 1




            @DimitriPavlov I found unwinding the functorial case in the relative category model to be highly involved. It is a large portion of the book of Dwyer-Hirschhorn-Kan-Smith. Usually you can punt the question and pop through to the hammock localization and then define it in terms of hom-wise simplicial holims, but in the Quasicategory case, the non-functorial definition amounts to nothing more than a terminal object of a slice over the diagram.
            – Harry Gindi
            13 hours ago






          • 1




            @HarryGindi: Sure, the unwinding can be involved, but notice that your original claim was not about the complexity of computations, but rather about the complexity of definitions, a different notion.
            – Dmitri Pavlov
            13 hours ago







          1




          1




          What's your def of homotopy (co)limits? I know it's discussed in the last two chapters of Hirschhorn's book by formulas. But his homotopy limits has not fully homotopy invariance—the objects involved in the diagrams should be fibrant in the model structure. But someone else will take functorial fibrant replacement before using the formulas, which I know is weakly equivalent to the right derived functor of $lim$ in nice cases (and someone always use this right derived functor as def of homotopy limits). Do you know what is the most "correct" and/or most accepted def?
          – Lao-tzu
          17 hours ago





          What's your def of homotopy (co)limits? I know it's discussed in the last two chapters of Hirschhorn's book by formulas. But his homotopy limits has not fully homotopy invariance—the objects involved in the diagrams should be fibrant in the model structure. But someone else will take functorial fibrant replacement before using the formulas, which I know is weakly equivalent to the right derived functor of $lim$ in nice cases (and someone always use this right derived functor as def of homotopy limits). Do you know what is the most "correct" and/or most accepted def?
          – Lao-tzu
          17 hours ago













          The difficulty of defining homotopy limits and colimits by universal property is one of the main motivations behind $infty$-categories. Without such a definition, all 'definitions' are actually nontrivially equivalent computations.
          – Harry Gindi
          16 hours ago




          The difficulty of defining homotopy limits and colimits by universal property is one of the main motivations behind $infty$-categories. Without such a definition, all 'definitions' are actually nontrivially equivalent computations.
          – Harry Gindi
          16 hours ago












          @Harry Gindi Is the notion of homotopy limits in a model category just a special case of limits for $infty$-categories (when taking the $infty$-category associated to the given model category) or there is some relation you can explain?
          – Lao-tzu
          15 hours ago




          @Harry Gindi Is the notion of homotopy limits in a model category just a special case of limits for $infty$-categories (when taking the $infty$-category associated to the given model category) or there is some relation you can explain?
          – Lao-tzu
          15 hours ago




          1




          1




          @DimitriPavlov I found unwinding the functorial case in the relative category model to be highly involved. It is a large portion of the book of Dwyer-Hirschhorn-Kan-Smith. Usually you can punt the question and pop through to the hammock localization and then define it in terms of hom-wise simplicial holims, but in the Quasicategory case, the non-functorial definition amounts to nothing more than a terminal object of a slice over the diagram.
          – Harry Gindi
          13 hours ago




          @DimitriPavlov I found unwinding the functorial case in the relative category model to be highly involved. It is a large portion of the book of Dwyer-Hirschhorn-Kan-Smith. Usually you can punt the question and pop through to the hammock localization and then define it in terms of hom-wise simplicial holims, but in the Quasicategory case, the non-functorial definition amounts to nothing more than a terminal object of a slice over the diagram.
          – Harry Gindi
          13 hours ago




          1




          1




          @HarryGindi: Sure, the unwinding can be involved, but notice that your original claim was not about the complexity of computations, but rather about the complexity of definitions, a different notion.
          – Dmitri Pavlov
          13 hours ago




          @HarryGindi: Sure, the unwinding can be involved, but notice that your original claim was not about the complexity of computations, but rather about the complexity of definitions, a different notion.
          – Dmitri Pavlov
          13 hours ago











          7














          In the language of $infty$-categories, which makes it a bit clearer, this is asking for the reflector (left adjoint) of the inclusion of a reflective subcategory to preserve filtered limits. This isn't true for ordinary categories, and there is also no reason to expect it to be true for $infty$-categories.



          Hirschhorn's Proposition 15.10.12 says that the homotopy limit of a tower of fibrations can be computed as the ordinary limit. Your modification asks for this homotopy limit to be preserved by the reflector (localization functor).






          share|cite|improve this answer
















          • 2




            As for a condition for this to hold, it's true if the $f_n$ are $M$-equivalences, but this makes the statement trivial, since it implies that the $X_n$ are $C$-local and forces the $p_n$ to be local fibrations (by the usual results about left Bousfield localization, being $C$-local is invariant under $M$-equivalence and $M$-fibrations between local objects are also local fibrations.)
            – Harry Gindi
            23 hours ago
















          7














          In the language of $infty$-categories, which makes it a bit clearer, this is asking for the reflector (left adjoint) of the inclusion of a reflective subcategory to preserve filtered limits. This isn't true for ordinary categories, and there is also no reason to expect it to be true for $infty$-categories.



          Hirschhorn's Proposition 15.10.12 says that the homotopy limit of a tower of fibrations can be computed as the ordinary limit. Your modification asks for this homotopy limit to be preserved by the reflector (localization functor).






          share|cite|improve this answer
















          • 2




            As for a condition for this to hold, it's true if the $f_n$ are $M$-equivalences, but this makes the statement trivial, since it implies that the $X_n$ are $C$-local and forces the $p_n$ to be local fibrations (by the usual results about left Bousfield localization, being $C$-local is invariant under $M$-equivalence and $M$-fibrations between local objects are also local fibrations.)
            – Harry Gindi
            23 hours ago














          7












          7








          7






          In the language of $infty$-categories, which makes it a bit clearer, this is asking for the reflector (left adjoint) of the inclusion of a reflective subcategory to preserve filtered limits. This isn't true for ordinary categories, and there is also no reason to expect it to be true for $infty$-categories.



          Hirschhorn's Proposition 15.10.12 says that the homotopy limit of a tower of fibrations can be computed as the ordinary limit. Your modification asks for this homotopy limit to be preserved by the reflector (localization functor).






          share|cite|improve this answer












          In the language of $infty$-categories, which makes it a bit clearer, this is asking for the reflector (left adjoint) of the inclusion of a reflective subcategory to preserve filtered limits. This isn't true for ordinary categories, and there is also no reason to expect it to be true for $infty$-categories.



          Hirschhorn's Proposition 15.10.12 says that the homotopy limit of a tower of fibrations can be computed as the ordinary limit. Your modification asks for this homotopy limit to be preserved by the reflector (localization functor).







          share|cite|improve this answer












          share|cite|improve this answer



          share|cite|improve this answer










          answered 23 hours ago









          Harry Gindi

          8,849676168




          8,849676168







          • 2




            As for a condition for this to hold, it's true if the $f_n$ are $M$-equivalences, but this makes the statement trivial, since it implies that the $X_n$ are $C$-local and forces the $p_n$ to be local fibrations (by the usual results about left Bousfield localization, being $C$-local is invariant under $M$-equivalence and $M$-fibrations between local objects are also local fibrations.)
            – Harry Gindi
            23 hours ago













          • 2




            As for a condition for this to hold, it's true if the $f_n$ are $M$-equivalences, but this makes the statement trivial, since it implies that the $X_n$ are $C$-local and forces the $p_n$ to be local fibrations (by the usual results about left Bousfield localization, being $C$-local is invariant under $M$-equivalence and $M$-fibrations between local objects are also local fibrations.)
            – Harry Gindi
            23 hours ago








          2




          2




          As for a condition for this to hold, it's true if the $f_n$ are $M$-equivalences, but this makes the statement trivial, since it implies that the $X_n$ are $C$-local and forces the $p_n$ to be local fibrations (by the usual results about left Bousfield localization, being $C$-local is invariant under $M$-equivalence and $M$-fibrations between local objects are also local fibrations.)
          – Harry Gindi
          23 hours ago





          As for a condition for this to hold, it's true if the $f_n$ are $M$-equivalences, but this makes the statement trivial, since it implies that the $X_n$ are $C$-local and forces the $p_n$ to be local fibrations (by the usual results about left Bousfield localization, being $C$-local is invariant under $M$-equivalence and $M$-fibrations between local objects are also local fibrations.)
          – Harry Gindi
          23 hours ago


















          draft saved

          draft discarded
















































          Thanks for contributing an answer to MathOverflow!


          • Please be sure to answer the question. Provide details and share your research!

          But avoid …


          • Asking for help, clarification, or responding to other answers.

          • Making statements based on opinion; back them up with references or personal experience.

          Use MathJax to format equations. MathJax reference.


          To learn more, see our tips on writing great answers.





          Some of your past answers have not been well-received, and you're in danger of being blocked from answering.


          Please pay close attention to the following guidance:


          • Please be sure to answer the question. Provide details and share your research!

          But avoid …


          • Asking for help, clarification, or responding to other answers.

          • Making statements based on opinion; back them up with references or personal experience.

          To learn more, see our tips on writing great answers.




          draft saved


          draft discarded














          StackExchange.ready(
          function ()
          StackExchange.openid.initPostLogin('.new-post-login', 'https%3a%2f%2fmathoverflow.net%2fquestions%2f319485%2flimit-of-weak-equivalences-in-a-bousfield-localization%23new-answer', 'question_page');

          );

          Post as a guest















          Required, but never shown





















































          Required, but never shown














          Required, but never shown












          Required, but never shown







          Required, but never shown

































          Required, but never shown














          Required, but never shown












          Required, but never shown







          Required, but never shown







          Popular posts from this blog

          Top Tejano songwriter Luis Silva dead of heart attack at 64

          ReactJS Fetched API data displays live - need Data displayed static

          政党